0% found this document useful (0 votes)
39 views14 pages

Final Review (Updated)

(1) The document provides a list of 12 practice problems for a final exam in MATH 2346 covering sets, logic, matrices, and their operations. (2) The problems include finding sets and their operations, writing logical statements using quantifiers, finding cartesian products of sets, solving systems of matrices, and properties of matrix operations. (3) Students are advised to review their lecture notes and examples from class to prepare for this exam focusing on core concepts from the first two units of the course.
Copyright
© © All Rights Reserved
We take content rights seriously. If you suspect this is your content, claim it here.
Available Formats
Download as PDF, TXT or read online on Scribd
0% found this document useful (0 votes)
39 views14 pages

Final Review (Updated)

(1) The document provides a list of 12 practice problems for a final exam in MATH 2346 covering sets, logic, matrices, and their operations. (2) The problems include finding sets and their operations, writing logical statements using quantifiers, finding cartesian products of sets, solving systems of matrices, and properties of matrix operations. (3) Students are advised to review their lecture notes and examples from class to prepare for this exam focusing on core concepts from the first two units of the course.
Copyright
© © All Rights Reserved
We take content rights seriously. If you suspect this is your content, claim it here.
Available Formats
Download as PDF, TXT or read online on Scribd
You are on page 1/ 14

MATH for EE and CE (MATH 2346) School of Mathematical and Statistical Sciences

Instructor: Dr. Ankush Goswami


Section 01
Final Review
April 30, 2023

Below is a list of problems for the final exam. Do check out the problems worked out in the examples in the lecture
notes and discussed in class.

1. Unit 1–2: Set, Logic, Matrices and their operations


(1) Let 𝐴 = [7], 𝐵 = {𝑥 ∈ 𝐴 : 𝑥 > 4}, 𝐶 = {𝑥 − 1 : 𝑥 ∈ 𝐵}. List the elements of (a) 𝐵, (b) 𝐶,
(c) 𝐵 ∩ 𝐶 (d) 𝐵 ∪ 𝐶 (e) 𝐴 − 𝐵 (f) 𝐵 − 𝐶 (g) 𝐶 − 𝐵.
Solution.
(a) 𝐵 = {5, 6, 7}, (b) 𝐶 = {4, 5, 6} (c) 𝐵 ∩ 𝐶 = {5, 6}, (d) 𝐵 ∪ 𝐶 = {4, 5, 6, 7}, (e)
𝐴 − 𝐵 = {1, 2, 3, 4} (f) 𝐵 − 𝐶 = {7}, (g) 𝐶 − 𝐵 = ∅.
(2) Write the statements (a) “∀𝑎 ∈ 𝐴 : ∃𝑏 ∈ 𝐵 : (3𝑎 > 𝑏 ⊻ 𝑎 + 𝑏 < 0)” and (b) “∀𝑎, 𝑏 ∈ 𝐴 :
∃𝑐 ∈ 𝐵 : ∀𝑑 ∈ 𝐴 : 𝑎𝑏 + 𝑏𝑑 < 3” in English.
Solution. (a) For all 𝑎 ∈ 𝐴, there is a 𝑏 ∈ 𝐵 such that either 3𝑎 > 𝑏 or 𝑎 + 𝑏 < 0 (not both).
(b) For all 𝑎, 𝑏 ∈ 𝐴, there is a 𝑐 ∈ 𝐵 such that for all 𝑑 ∈ 𝐴, 𝑎𝑏 + 𝑏𝑑 < 3.
(3) Write the following statements using quantifiers. (a) Every real number is equal to itself, (b)
There is a real number 𝑥 such that for all real numbers 𝑦, we have 𝑥 + 𝑦 = 0, (c) For all 𝜖 > 0,
there is a 𝛿 > 0 such that for all real numbers 𝑥, if 𝑥0 − 𝛿 < 𝑥 < 𝑥0 + 𝛿, then | 𝑓 (𝑥) − 𝑎| < 𝜖.
Solution. (a) ∀𝑥 ∈ R : 𝑥 = 𝑥. (b) ∃𝑥 ∈ R : ∀𝑦 ∈ R : 𝑥 + 𝑦 = 0. (c) ∀𝜖 > 0 : ∃𝛿 > 0 : ∀𝑥 ∈
R : 𝑥 0 − 𝛿 < 𝑥 < 𝑥0 + 𝛿 ⇒ | 𝑓 (𝑥) − 𝑎| < 𝜖.
(4) Find the following cartesian products. (a) 𝐴 × 𝐵 with 𝐴 = {2, 3, 4}, 𝐵 = {7, 8}, (b)
[2] × [3], (c) 𝐴 × 𝐵 × 𝐶 with 𝐴 = [3] − {1}, 𝐵 = [3] ∩ [6], 𝐶 = [2].
Solution. (a) 𝐴 × 𝐵 = {2, 3, 4} × {7, 8} = {(2, 7), (2, 8), (3, 7), (3, 8), (4, 7), (4, 8)}.
(b) [2] × [3] = {1, 2} × {1, 2, 3} = {(1, 1), (1, 2), (1, 3), (2, 1), (2, 2), (2, 3)}.
(c) 𝐴 × 𝐵 = {2, 3} × {1, 2, 3} = {(2, 1), (2, 2), (2, 3), (3, 1), (3, 2), (3, 3)}. Thus,
𝐴 × 𝐵 × 𝐶 = ( 𝐴 × 𝐵) × 𝐶 = {(2, 1), (2, 2), (2, 3), (3, 1), (3, 2), (3, 3)} × {1, 2}
= {(2, 1, 1), (2, 2, 1), (2, 3, 1), (3, 1, 1), (3, 2, 1), (3, 3, 1),
(2, 1, 2), (2, 2, 2), (2, 3, 2), (3, 1, 2), (3, 2, 2), (3, 3, 2)}.
 
2 5
(5) Consider the matrix 𝐴 = . Find the unique 𝑥, 𝑦 ∈ R such that 𝐴2 = 𝑥 𝐴 + 𝑦𝐼.
3 1
Solution. We have
      
2 2 5 2 5 19 15 2𝑥 + 𝑦 5𝑥
𝐴 = = , 𝑥 𝐴 + 𝑦𝐼 = .
3 1 3 1 9 16 3𝑥 𝑥+𝑦
So, we have 2𝑥 + 𝑦 = 19, 5𝑥 = 15, 3𝑥 = 9, 𝑥 + 𝑦 = 16, solving which yields 𝑥 = 3, 𝑦 = 13.
 
𝑎 𝑏
(6) Given the matrix 𝐴 = . Show that 𝐴2 − (𝑎 + 𝑑) 𝐴 + (𝑎𝑑 − 𝑏𝑐)𝐼 = 0.
𝑐 𝑑
Solution. We have
    2   2 
2 𝑎 𝑏 𝑎 𝑏 𝑎 + 𝑏𝑐 𝑎𝑏 + 𝑏𝑑 𝑎 + 𝑎𝑑 𝑎𝑏 + 𝑏𝑑
𝐴 = = , (𝑎 + 𝑑) 𝐴 = ,
𝑐 𝑑 𝑐 𝑑 𝑎𝑐 + 𝑐𝑑 𝑏𝑐 + 𝑑 2 𝑎𝑐 + 𝑐𝑑 𝑎𝑑 + 𝑑 2
 
𝑎𝑑 − 𝑏𝑐 0
(𝑎𝑑 − 𝑏𝑐)𝐼 = .
0 𝑎𝑑 − 𝑏𝑐
Thus,
𝐴2 − (𝑎 + 𝑑) 𝐴 + (𝑎𝑑 − 𝑏𝑐)𝐼
 2   2     
𝑎 + 𝑏𝑐 𝑎𝑏 + 𝑏𝑑 𝑎 + 𝑎𝑑 𝑎𝑏 + 𝑏𝑑 𝑎𝑑 − 𝑏𝑐 0 0 0
= − + = .
𝑎𝑐 + 𝑐𝑑 𝑏𝑐 + 𝑑 2 𝑎𝑐 + 𝑐𝑑 𝑎𝑑 + 𝑑 2 0 𝑎𝑑 − 𝑏𝑐 0 0
   
1 −1 3 4
(7) For the matrices 𝐴 = ,𝐵 = . Determine if 𝐴𝐵 = 𝐵𝐴.
2 3 2 7
Solution. We have
         
1 −1 3 4 1 −3 3 4 1 −1 11 9
𝐴𝐵 = = , 𝐵𝐴 = = .
2 3 2 7 12 29 2 7 2 3 16 19
Clearly, 𝐴𝐵 ≠ 𝐵𝐴.
(8) Let 𝐴𝐵 = 𝐵𝐴. Show that (a) ( 𝐴 − 𝜆𝐼)(𝐵 − 𝜆𝐼) = (𝐵 − 𝜆𝐼)( 𝐴 − 𝜆𝐼) for 𝜆 ∈ R.
(b) 𝐴2 𝐵2 = 𝐵2 𝐴2 .
Solution. (a) We have
( 𝐴 − 𝜆𝐼)(𝐵 − 𝜆𝐼) = 𝐴𝐵 − 𝜆𝐴 − 𝜆𝐵 + 𝜆2 𝐼
= 𝐵𝐴 − 𝜆𝐵 − 𝜆𝐴 + 𝜆2 𝐼 (using 𝐴𝐵 = 𝐵𝐴)
= (𝐵 − 𝜆𝐼)( 𝐴 − 𝜆𝐼).
(b) We have
𝐴2 𝐵2 = 𝐴𝐴𝐵𝐵 = 𝐴( 𝐴𝐵)𝐵, (using associativity)
= 𝐴(𝐵𝐴)𝐵, (using 𝐴𝐵 = 𝐵𝐴)
= 𝐴𝐵𝐴𝐵 = ( 𝐴𝐵)( 𝐴𝐵), (using associativity)
= (𝐵𝐴)(𝐵𝐴), (using 𝐴𝐵 = 𝐵𝐴)
= 𝐵( 𝐴𝐵) 𝐴, (using associativity)
= 𝐵𝐵𝐴𝐴, (using 𝐴𝐵 = 𝐵𝐴)
= 𝐵 2 𝐴2 .

(9) If ( 𝐴 + 𝐵) 2 = 𝐴2 + 2𝐴𝐵 + 𝐵2 . Show that 𝐴𝐵 = 𝐵𝐴.


Solution. We have
( 𝐴 + 𝐵) 2 = ( 𝐴 + 𝐵)( 𝐴 + 𝐵) = 𝐴2 + 𝐴𝐵 + 𝐵𝐴 + 𝐵2 .
Thus, ( 𝐴 + 𝐵) 2 = 𝐴2 + 2𝐴𝐵 + 𝐵2 ⇔ 𝐴2 + 𝐴𝐵 + 𝐵𝐴 + 𝐵2 = 𝐴2 + 2𝐴𝐵 + 𝐵2 ⇔ 𝐴𝐵 = 𝐵𝐴.
2
(10) Let 𝐴 ∈ 𝑀𝑛 (R) be such that 𝐴2 = 𝐼. Show that the matrices 𝐵 = 12 (𝐼 + 𝐴) and 𝐶 = 12 (𝐼 − 𝐴)
satisfy 𝐵2 = 𝐵 and 𝐶 2 = 𝐶.
Solution. We have
 2
2 1 1
𝐵 = (𝐼 + 𝐴) = (𝐼 + 𝐴)(𝐼 + 𝐴)
2 4
1  1
= 𝐼 + 2𝐴 + 𝐴2 = (𝐼 + 2𝐴 + 𝐼), (using 𝐴2 = 𝐼)
4 4
1
= (𝐼 + 𝐴).
2
In a similar way, it follows that 𝐶 2 = 𝐶.
(11) Let 𝐴, 𝐵, 𝐶 ∈ 𝑀𝑛 (R). Is it true that (a) 𝐴𝐶 = 𝐵𝐶 ⇒ 𝐴 = 𝐵 or (b) 𝐶 𝐴 = 𝐶𝐵 ⇒ 𝐴 = 𝐵? If
not, under what condition(s) does this property hold?
Solution. Properties (a) or (b) are called cancellation property which holds only when
𝐶 ∈ 𝐺 𝐿 𝑛 (R) (the set of all 𝑛 × 𝑛 invertible matrices). (a) Assume 𝐴𝐶 = 𝐵𝐶. Then with
𝐶 ∈ 𝐺 𝐿 𝑛 (R), we have

𝐴 = ( 𝐴𝐶)𝐶 −1 (using associativity)


= (𝐵𝐶)𝐶 −1 (using 𝐴𝐶 = 𝐵𝐶)
= 𝐵.

In a similar way, we can show (b).


(12) Use matrix inverse to solve the systems:
)
𝑥 + 3𝑦 = 4,
 𝑎𝑥 − 𝑦 = 1 − 𝑎,
(𝑖) , (𝑖𝑖) 𝑎 is a parameter
2𝑥 + 𝑦 = 3. 𝑥 − 𝑎𝑦 = 𝑎 − 𝑎 2 .
.
Solution. Recall that the system 𝐴x = b has a unique solution if 𝐴 is invertible, in which
case, the solution is given by x = 𝐴−1 b.
(i) The system of equations is equivalent to
     
1 3 𝑥 4
= . (1.1)
2 1 𝑦 3
| {z } |{z} |{z}
𝐴 x b

Computing determinant, we find | 𝐴| = 1 · 1 − 2 · 3 = −5 ≠ 0. So, 𝐴 is invertible, and


   
−1 1 1 −3 −1/5 3/5
𝐴 = = .
−5 −2 1 2/5 −1/5

Thus, the unique solution of (1.1) is given by


      
𝑥 −1 −1/5 3/5 4 1
x= =𝐴 b= = ⇒ 𝑥 = 𝑦 = 1.
𝑦 2/5 −1/5 3 1
3
(ii) The system of equations is equivalent to
     
𝑎 −1 𝑥 1−𝑎
= (1.2)
1 −𝑎 𝑦 𝑎 − 𝑎2
| {z } |{z} | {z }
𝐴 x b

In this case, | 𝐴| = 𝑎(−𝑎) − (−1)1 = 1 − 𝑎 2 . Thus, we see that | 𝐴| = 0 ⇔ 𝑎 = ±1.


Case 1: If 𝑎 ≠ ±1. In this case | 𝐴| ≠ 0. Thus, we have a unique solution of (1.2) given by
       
𝑥 −1 1 −𝑎 1 1 − 𝑎 (1 − 𝑎) −𝑎 1 1
x= =𝐴 b= 2 =
𝑦 1 − 𝑎 2 −1 𝑎 𝑎 − 𝑎 1 − 𝑎 2 −1 𝑎 𝑎
   
1 0 0
=
−(1 − 2 ) = 𝑎 − 1 ⇒ 𝑥 = 0, 𝑦 = 𝑎 − 1.
1+𝑎 𝑎
Case 2: 𝑎 = 1. In this case, the both the equations reduce to
𝑥−𝑦=0⇔𝑦=𝑥
which has infinitely many solutions.
Case 3: If 𝑎 = −1. In this case, the given equations reduce to
𝑥 + 𝑦 = −2 ⇔ 𝑦 = −𝑥 − 2
which also leads to infinitely many solutions.
(13) If 𝐴, 𝐵 ∈ 𝑀𝑛 (R) are non-singular (invertible) matrices, show that 𝐴𝐵 is also non-singular,
and ( 𝐴𝐵) −1 = 𝐵−1 𝐴−1 .
Solution. Since 𝐴 and 𝐵 are non-singular, there are matrices 𝐶, 𝐷 ∈ 𝐺 𝐿 𝑛 (R) such that
𝐴𝐶 = 𝐶 𝐴 = 𝐼, and 𝐵𝐷 = 𝐷𝐵 = 𝐼 (recall that 𝐶 and 𝐷 are called inverses of 𝐴 and 𝐵
respectively). Now,
( 𝐴𝐵)(𝐷𝐶) = 𝐴(𝐵𝐷)𝐶 = 𝐴(𝐼)𝐶, (using 𝐵𝐷 = 𝐼)
= 𝐴𝐶 = 𝐼, (using 𝐴𝐶 = 𝐼).
Similarly, it follows that (𝐷𝐶)( 𝐴𝐵) = 𝐼. This shows that 𝐴𝐵 is non-singular since there is
a matrix 𝑀 = 𝐷𝐶 such that 𝑀 ( 𝐴𝐵) = ( 𝐴𝐵)𝑀 = 𝐼. Next, we have
(𝐵−1 𝐴−1 ) 𝐴𝐵 = 𝐵−1 ( 𝐴−1 𝐴)𝐵 = 𝐵−1 (𝐼)𝐵 = 𝐵−1 𝐵 = 𝐼,
)

−1 −1 −1 −1 −1 −1
⇒ ( 𝐴𝐵) −1 = 𝐵−1 𝐴−1 .
( 𝐴𝐵)(𝐵 𝐴 ) = 𝐴(𝐵𝐵 ) 𝐴 = 𝐴(𝐼) 𝐴 = 𝐴𝐴 = 𝐼.

(14) If 𝐴 ∈ 𝑀𝑛 (R) with 𝐴3 = 0, show that (𝐼 − 𝐴) is non-singular with (𝐼 − 𝐴) −1 = 𝐼 + 𝐴 + 𝐴2 .


Solution. Since 𝐴3 = 0, we have
)
(𝐼 − 𝐴)(𝐼 + 𝐴 + 𝐴2 ) = 𝐼 + 𝐴 + 𝐴2 − 𝐴 − 𝐴2 − 𝐴3 = 𝐼,
2 2 2 3
⇒ (𝐼 − 𝐴) −1 = 𝐼 + 𝐴 + 𝐴2 .
(𝐼 + 𝐴 + 𝐴 )(𝐼 − 𝐴) = 𝐼 + 𝐴 + 𝐴 − 𝐴 − 𝐴 − 𝐴 = 𝐼.

(15) Let 𝐴 ∈ 𝑀𝑛 (R) satisfies 𝐴2 + 𝐴 + 𝐼 = 0, show that 𝐴 is non-singular and 𝐴−1 = 𝐴2 .


Solution. We have
𝐴2 + 𝐴 + 𝐼 = 0 ⇔ 𝐴(−𝐼 − 𝐴) = 𝐼 ⇔ 𝐴−1 = −𝐼 − 𝐴 = 𝐴2 (since − 𝐴 − 𝐼 = 𝐴2 ).
4
(16) Let 𝐴, 𝐵 ∈ 𝑀𝑛 (R). If 𝐴, 𝐵, 𝐴𝐵 are symmetric, then show that 𝐴𝐵 = 𝐵𝐴.
Solution. Recall that a matrix 𝑀 is symmteric if 𝑀 𝑇 = 𝑀. We are given 𝐴𝑇 = 𝐴, 𝐵𝑇 =
𝐵, ( 𝐴𝐵)𝑇 = 𝐴𝐵. Now,
𝐴𝐵 = ( 𝐴𝐵)𝑇 (since 𝐴𝐵 is symmteric)
= 𝐵𝑇 𝐴𝑇 (since ( 𝐴𝐵)𝑇 = 𝐵𝑇 𝐴𝑇 )
= 𝐵𝐴 (since 𝐴, 𝐵 are symmteric).
(17) A matrix 𝑀 ∈ 𝑀𝑛 (R) is said to be skew-symmetric if 𝑀 𝑇 = −𝑀. Let 𝐴, 𝐵 ∈ 𝑀𝑛 (R) be
such that 𝐴 is symmetric and 𝐵 is skew-symmetric. Show that 𝐴2 𝐵𝐴2 is skew-symmetric.
Solution. Let us put 𝑀 = 𝐴2 𝐵𝐴2 . We have
𝑀 𝑇 = ( 𝐴2 𝐵𝐴2 )𝑇 = ( 𝐴2 )𝑇 ( 𝐴2 𝐵)𝑇 = ( 𝐴2 )𝑇 (𝐵𝑇 ( 𝐴2 )𝑇 ), (using (𝐶𝐷)𝑇 = 𝐷 𝑇 𝐶 𝑇 )
= ( 𝐴𝑇 ) 2 (𝐵𝑇 ( 𝐴𝑇 ) 2 ) = 𝐴2 (−𝐵𝐴2 ), (using 𝐴𝑇 = 𝐴, 𝐵𝑇 = −𝐵)
= −𝐴2 𝐵𝐴2 = −𝑀.

(18) If 𝐴 ∈ 𝑀𝑛 (R) is symmetric and non-singular, then 𝐴−1 is symmetric.


Solution. We have
( 𝐴𝑇 )( 𝐴−1 )𝑇 = ( 𝐴−1 𝐴)𝑇 = 𝐼 𝑇 = 𝐼, (using ( 𝐴𝐵)𝑇 = 𝐵𝑇 𝐴𝑇 )
( 𝐴−1 )𝑇 ( 𝐴𝑇 ) = ( 𝐴𝐴−1 )𝑇 = 𝐼 𝑇 = 𝐼, (using ( 𝐴𝐵)𝑇 = 𝐵𝑇 𝐴𝑇 )

The above shows that ( 𝐴𝑇 ) −1 = ( 𝐴−1 )𝑇 . Since 𝐴𝑇 = 𝐴, this shows that 𝐴−1 = ( 𝐴−1 )𝑇 .
Thus, 𝐴−1 is symmetric.

(19) Let 𝐴, 𝑃 ∈ 𝑀𝑛 (R), show that 𝐴 is symmetric implies 𝐵 = 𝑃𝑇 𝐴𝑃 is symmetric.


Solution. We have 𝐴 is symmetric. So, 𝐴𝑇 = 𝐴. Thus,
𝐵𝑇 = (𝑃𝑇 𝐴𝑃)𝑇 = 𝑃𝑇 (𝑃𝑇 𝐴)𝑇 = 𝑃𝑇 ( 𝐴𝑇 (𝑃𝑇 )𝑇 )
= 𝑃𝑇 ( 𝐴𝑃), (using (𝑃𝑇 )𝑇 = 𝑃, 𝐴𝑇 = 𝐴)
= 𝑃𝑇 𝐴𝑃 = 𝐵.

(20) Let 𝐴, 𝐵 ∈ 𝑀𝑛 (R) such that 𝐴2 = 𝐼 and 𝐵2 = 𝐵. Show that (2𝐵 − 𝐼) 2 = 𝐼 and ( 𝐴 + 𝐼) 2 =
2( 𝐴 + 𝐼).
Solution. We have
(2𝐵 − 𝐼) 2 = (2𝐵 − 𝐼)(2𝐵 − 𝐼) = 4𝐵2 − 2𝐵 − 2𝐵 + 𝐼 = 4𝐵 − 4𝐵 + 𝐼 = 𝐼, (using 𝐵2 = 𝐵)
and
( 𝐴 + 𝐼) 2 = ( 𝐴 + 𝐼)( 𝐴 + 𝐼) = 𝐴2 + 𝐴 + 𝐴 + 𝐼 = 𝐼 + 2𝐴 + 𝐼 = 2( 𝐴 + 𝐼), (using 𝐴2 = 𝐼).
(21) Let 𝑥 represent a student in this class, and 𝑃(𝑥) and 𝑄(𝑥) be the following predicates:
𝑃(𝑥) : 𝑥 has a background in python programming,
𝑄(𝑥) : 𝑥 has a background in 𝐶 ++ coding.
5
Then write the statement “There exists a student in this class that has a background in either
python or 𝐶 ++ coding but not both” using predicates.
Solution. The above statement is equivalent to
∃𝑥 : 𝑃(𝑥) ⊻ 𝑄(𝑥).

(22) Let ( 𝐴 − 𝐵) 2 = 𝐴2 − 2𝐴𝐵 + 𝐵2 . Show that 𝐴𝐵 = 𝐵𝐴.


Solution. Foil the left-hand side of the given expression
( 𝐴 − 𝐵) 2 = ( 𝐴 − 𝐵)( 𝐴 − 𝐵) = 𝐴2 − 𝐴𝐵 − 𝐵𝐴 + 𝐵2

which equals 𝐴2 − 2𝐴𝐵 + 𝐵2 . Cancelling out common terms, we arrive at 𝐴𝐵 = 𝐵𝐴.


(23) (Traffic Flow) Consider the figure below near the university which shows the number of
cars (per hour) entering and leaving the junctions 𝐴 and 𝐵 at the W University Ave through
Sugar Rd and Jackson Rd, respectively.

𝑥 → the number of cars (per hour) entering (exiting) the junction 𝐴 (resp. 𝐵),
𝑦 → the number of cars (per hour) entering Sugar and Jackson roads.
Find the values of 𝑥 and 𝑦, assuming that same number of cars enter and exit a junction.
The numbers (100, 200 and 250) indicate number of cars entering or exiting a junction per
hour.
Solution. We get the following system of equations:
𝑥 + 𝑦 = 200 + 250 = 450,

At A :
At B : 𝑦 + 200 = 𝑥 + 100 ⇔ 𝑥 − 𝑦 = 100.
Solving the system yields 𝑥 = 275, 𝑦 = 175.
6
(24) What is the parity of the permutation 𝜎 = (1, 4, 3, 2) and 𝛿 = (2, 3, 4, 1)?
Solution. We can construct 𝜎 using transpositions as follows:

(1, 2, 3, 4) → (1, 4, 3, 2) = 𝜎
(1, 2, 3, 4) → (2, 1, 3, 4) → (2, 4, 3, 1) → (2, 3, 4, 1) = 𝛿.

Thus, 𝜎 is odd and 𝛿 is odd.


(25) Calculate the determinant of

𝑎 𝑏 𝑐

𝑐 𝑎 𝑏

𝑏 𝑎 𝑐

Solution. We have

𝐶1→𝐶2+𝐶3 𝑎 + 𝑏 + 𝑐 𝑏 𝑐
𝑎 𝑏 𝑐 1 𝑏 𝑐

𝑐 𝑎 𝑏 −−−−−−−−−→ 𝑎 + 𝑏 + 𝑐 𝑎 𝑏 = (𝑎 + 𝑏 + 𝑐) 1 𝑎 𝑏

𝑏 𝑎 𝑐 𝑎+𝑏+𝑐 𝑎 𝑐 1 𝑎 𝑐

1 𝑏 𝑐
𝑅2→𝑅2−𝑅1 𝑎−𝑏 𝑏−𝑐
−−−−−−−−−→ (𝑎 + 𝑏 + 𝑐) 0 𝑎 − 𝑏 𝑏 − 𝑐 = (𝑎 + 𝑏 + 𝑐) = −(𝑎 + 𝑏 + 𝑐)(𝑎 − 𝑏)(𝑏 − 𝑐).
𝑅3→𝑅3−𝑅1 0 𝑎−𝑏 𝑎−𝑏 0
0

(26) Calculate the following determinant using cofactor expansion:



1 0 3

2 1 0

0 3 1

Solution. We perform cofactor expansion along the first column to get



1 0 3
2 1 0 = 1 · 1 0 − 2 · 0 3 = 1 · (1 − 0) − 2 · (0 − 9) = 1 + 18 = 19.

3 1 3 1
0 3 1
   
−1 −1 5 0 1 0
(27) If 𝐵 𝐴 = and 𝐵 = , find 𝐴.
2 −1 3 1
Solution. We have
    
−1 −1 −1 1 0 5 0 5 0
𝐴 = 𝐵(𝐵 𝐴 ) = = .
3 1 2 −1 17 −1
  −1    
5 0 1 −1 0 1/5 0
Thus 𝐴 = = = .
17 −1 (−5) −17 5 17/5 −1
(28) Use Cramer’s rule to solve the following system of equations.

𝑥 + 2𝑦 − 𝑧 = 0 


2𝑥 − 𝑦 + 3𝑧 = 0

𝑥+𝑦+𝑧=2  
7
Solution. The given system of equations is equivalent to the following matrix equation:
1 2 −1 𝑥  0
     
2 −1 3   𝑦  = 0 .
     
1 1 1   𝑧  2
     
| {z } |{z} |{z}
𝐴 x b

Let us put
0 2 −1 1 0 −1 1 2 0
     
𝐴1 (b) = 0 −1 3  , 𝐴2 (b) = 2 0 3  , 𝐴3 (b) = 2 −1 0
2 1 1  1 2 1  1 1 2
     
By calculating the determinants (using cofactor expansions), we find

| 𝐴| = −5, | 𝐴1 (b)| = 10, | 𝐴2 (b)| = −10, | 𝐴3 (b)| = −10.

Thus, by Cramer’s rule, we find


| 𝐴1 (b)| 10
𝑥 = = = −2,
| 𝐴| −5
| 𝐴2 (b)| −10
𝑦 = = = 2,
| 𝐴| −5
| 𝐴3 (b)| −10
𝑧 = = = 2.
| 𝐴| −5
(29) Try the problems in Test 2 Review from problem 10 onwards.

2. Graph Theory
(30) For the following graphs, list 𝑉 (𝐺), 𝐸 (𝐺) and the values of the incidence mapping 𝜓𝐺 .

8
Solution. For the first graph, 𝑉 (𝐺) = {𝑣 1 , 𝑣 2 , 𝑣 3 }, 𝐸 (𝐺) = {𝑒 1 , 𝑒 2 , 𝑒 3 } and 𝜓𝐺 (𝑒 1 ) =
{𝑣 1 , 𝑣 2 }, 𝜓𝐺 (𝑒 2 ) = {𝑣 1 , 𝑣 3 }, 𝜓𝐺 (𝑒 3 ) = {𝑒 3 }.
For the second graph, 𝑉 (𝐺) = {𝑣 1 , 𝑣 2 , 𝑣 3 , 𝑣 4 }, 𝐸 (𝐺) = {𝑒 1 , 𝑒 2 , 𝑒 3 , 𝑒 4 , 𝑒 5 , 𝑒 6 } and
𝜓𝐺 (𝑒 1 ) = {𝑣 1 , 𝑣 2 }, 𝜓𝐺 (𝑒 2 ) = {𝑣 2 , 𝑣 3 }, 𝜓𝐺 (𝑒 3 ) = {𝑣 2 , 𝑣 3 }, 𝜓𝐺 (𝑒 4 ) = {𝑣 1 , 𝑣 3 }, 𝜓𝐺 (𝑒 5 ) =
{𝑣 3 , 𝑣 4 }, 𝜓𝐺 (𝑒 6 ) = {𝑣 1 , 𝑣 4 }.

(31) Is it possible to construct a graph with 7 vertices of degree 3 and 2 vertices of degree 2?
Solution. Let 𝑉 (𝐺) = {𝑣 1 , 𝑣 2 , 𝑣 3 , 𝑣 4 , 𝑣 5 , 𝑣 6 , 𝑣 7 , 𝑢 1 , 𝑢 2 } where 𝑑 (𝑣 𝑖 ) = 3, 1 ≤ 𝑖 ≤ 7 and
𝑑 (𝑢 1 ) = 𝑑 (𝑢 2 ) = 2. Here 𝑑 (𝑤) denotes the degree of the vertex 𝑤. By the handshaking
lemma, we have

1 ∑︁ 1
|𝐸 (𝐺)| = 𝑑 (𝑤) = (𝑑 (𝑣 1 ) + · · · + 𝑑 (𝑣 7 ) + 𝑑 (𝑢 1 ) + 𝑑 (𝑢 2 ))
2 2
𝑤∈𝑉 (𝐺)

1© ª 1 25
= ­­3 + · · · + 3 +2 + 2®® = (7 × 3 + 4) = .
2 | {z } 2 2
« 7 times ¬

Since the number of edges |𝐸 (𝐺)| cannot be fraction, the graph with the required properties
in question is not possible.

(32) Let 𝐺 be a graph with 10 vertices such that 𝛿(𝐺) = Δ(𝐺) = 2. How many edges does 𝐺
have?
Solution. Let 𝑢 be any vertex in the graph 𝐺. Then we know (from definition of 𝛿(𝐺) and
Δ(𝐺)) that

𝛿(𝐺) ≤ 𝑑 (𝑢) ≤ Δ(𝐺) ⇒ 𝑑 (𝑢) = 2.


9
By the handshaking lemma, we have

1 ∑︁ 1© ª
|𝐸 (𝐺)| = 𝑑 (𝑢) = ­­2 + 2 + · · · + 2®® = 10.
2 2 | {z }
𝑢∈𝑉 (𝐺)
« 10 times ¬
(33) Let 𝐺 be a graph with Δ(𝐺) = 4. Show that |𝐸 (𝐺)| ≤ 2|𝑉 (𝐺)|.
Solution. Let 𝑢 be any vertex of 𝐺. Then Δ(𝐺) = 4 implies 𝑑 (𝑢) ≤ 4. By the handshaking
lemma, we have
1 ∑︁ 1 ∑︁ ∑︁
|𝐸 (𝐺)| = 𝑑 (𝑢) ≤ 4=2 1 = 2|𝑉 (𝐺)|.
2 2
𝑢∈𝑉 (𝐺) 𝑢∈𝑉 (𝐺) 𝑢∈𝑉 (𝐺)

(34) A graph with 4 edges has a vertex with degree 4, a vertex with degree 1 and one more
vertex. What is the degree of the third vertex?
Solution. For the graph 𝐺, let 𝑉 (𝐺) = {𝑢 1 , 𝑢 2 , 𝑢 3 } with 𝑑 (𝑢 1 ) = 4, 𝑑 (𝑢 2 ) = 1 and
|𝐸 (𝐺)| = 4. By the handshaking lemma, we have
1 ∑︁ 1
|𝐸 (𝐺)| = 𝑑 (𝑢) = (𝑑 (𝑢 1 ) + 𝑑 (𝑢 2 ) + 𝑑 (𝑢 3 ))
2 2
𝑢∈𝑉 (𝐺)
1
⇔4= (4 + 1 + 𝑑 (𝑢 3 )) ⇔ 8 = 5 + 𝑑 (𝑢 3 ) ⇔ 𝑑 (𝑢 3 ) = 3.
2
(35) Find 𝛿(𝐺) and Δ(𝐺) for each of the graphs 𝐺 = 𝐾5 , 𝐾5,7 , 𝑃4 , 𝐶6 . Find |𝑉 (𝐺)| and |𝐸 (𝐺)|
for each of these graphs.
Solution. Recall that 𝐾5 is the unique complete graph with 5 vertices. So, |𝑉 (𝐾5 )| = 5 and
𝐸 (𝐾5 ) = 9. Since 𝐾5 is complete, every vertex in 𝐾5 is connected to the other 4 vertices by
an edge, so 𝛿(𝐾5 ) = Δ(𝐾5 ) = 4.
Recall that 𝐾5,7 is the unique complete bipartite graph with vertex partition 𝑉1 and 𝑉2
(𝑉 (𝐾5,7 ) = 𝑉1 ∪ 𝑉2 ) such that |𝑉1 | = 5 and |𝑉2 | = 7. Also, each vertex in 𝑉1 is connected to
every vertex in 𝑉2 . Thus, |𝑉 (𝐾5,7 )| = 5 + 7 = 12 and |𝐸 (𝐾5,7 )| = 5 × 7 = 35. Next, for any
vertex 𝑢 ∈ 𝑉1 , we have 𝛿(𝑢) = Δ(𝑢) = 7 and for 𝑣 ∈ 𝑉2 , we have 𝛿(𝑣) = Δ(𝑣) = 5.
Recall that 𝑃4 is the path graph with 4 vertices. So, |𝑉 (𝑃4 )| = 4 and |𝐸 (𝑃4 )| = 3. Also,
if 𝑉 (𝑃4 ) = {𝑢 1 , 𝑢 2 , 𝑢 3 , 𝑢 4 } with 𝑢 1 (resp. 𝑢 4 ) be the initial (resp. terminal) vertex of 𝑃4 ,
then 𝑑 (𝑢 1 ) = 𝑑 (𝑢 4 ) = 1 and 𝑑 (𝑢 2 ) = 𝑑 (𝑢 3 ) = 2. Thus, 𝛿(𝑃4 ) = 1 and Δ(𝑃4 ) = 2.
Recall that 𝐶6 is the cycle graph with 6 vertices. Then |𝑉 (𝐶6 )| = 6 and |𝐸 (𝐶6 )| = 6.
Also, we have 𝛿(𝐶6 ) = Δ(𝐶6 ) = 2.

(36) For positive integers 𝑎, 𝑏, show that 𝐾𝑎,𝑏 is regular if and only if 𝑎 = 𝑏.
Solution. Using the ideas from the last problem, we can conclude that if 𝑉1 and 𝑉2 are
vertex partitions (𝑉 (𝐾𝑎,𝑏 ) = 𝑉1 ∪ 𝑉2 ), then |𝑉1 | = 𝑎 and |𝑉2 | = 𝑏. Then for any vertex
𝑢 ∈ 𝑉1 , we have 𝑑 (𝑢) = 𝑏 and for any vertex 𝑣 ∈ 𝑉2 , 𝑑 (𝑣) = 𝑎. Thus, for 𝐾𝑎,𝑏 to be regular
(all vertices have same degree), we must have 𝑑 (𝑢) = 𝑑 (𝑣), that is, 𝑎 = 𝑏. Also, if 𝑎 = 𝑏,
then one can easily check that the graph 𝐾𝑎,𝑎 is regular.

(37) Let 𝐺 be a bipartite graph with bipartition 𝑉 (𝐺) = 𝑉1 ∪ 𝑉2 . If |𝑉1 | = 𝑎 and |𝑉2 | = 𝑎 + 2.
Show that |𝐸 (𝐺)| ≤ 𝑎 2 + 2𝑎.
Solution. Note that 𝐺 may not be a complete bipartite graph. So, for any vertex 𝑢 ∈ 𝑉1 ,
10
we have 𝑑 (𝑢) ≤ 𝑎 + 2 and for any 𝑣 ∈ 𝑉2 , we have 𝑑 (𝑣) = 𝑎. By the handshaking lemma,
we have
!
1 ∑︁ 1 ∑︁ ∑︁
|𝐸 (𝐺)| = 𝑑 (𝑤) = 𝑑 (𝑢) + 𝑑 (𝑣)
2 2 𝑢∈𝑉
𝑤∈𝑉 (𝐺) 1 𝑣∈𝑉2
!
1 ∑︁ ∑︁
≤ (𝑎 + 2) 1+𝑎 1
2 𝑢∈𝑉 𝑢∈𝑉
1 2

1
= ((𝑎 + 2)|𝑉1 | + 𝑎|𝑉2 |)
2
1
= ((𝑎 + 2)𝑎 + 𝑎(𝑎 + 2)) = 𝑎 2 + 2𝑎.
2
(38) Let us the consider the complete bipartite graph with 3 vertices with vertex partition
𝑉 (𝐺) = 𝑉1 ∪𝑉2 with 𝑉1 = {𝑎, 𝑏, 𝑐} and 𝑉2 = {𝑑, 𝑒, 𝑓 }. Find 𝐺 [{𝑎, 𝑏, 𝑑}] and 𝐺 − {𝑑, 𝑒, 𝑓 }.

Solution. Recall that if 𝑉0 ⊂ 𝑉 (𝐺), then 𝐺 [𝑉0 ] is the vertex induced subgraph with
vertex set 𝑉0 . Thus, 𝐺 [{𝑎, 𝑏, 𝑑}] would be

and the graph 𝐺 − {𝑑, 𝑒, 𝑓 } = 𝐺 [𝑉 (𝐺) − {𝑑, 𝑒, 𝑓 }] = 𝐺 [{𝑎, 𝑏, 𝑐}] would be

(39) Consider the graph 𝐺 given by


11
and the following walks: 𝑤 1 = (𝑎, 𝑒 1 , 𝑎, 𝑒 2 , 𝑏, 𝑒 3 , 𝑐) and 𝑤 2 = (𝑎, 𝑒 2 , 𝑏, 𝑒 4 , 𝑐). Deter-
mine if these walks are path or trail.
Solution. Note that each of the walks is an alternating sequence of vertices and edges. 𝑤 1
is a walk where the edges are distinct, but the vertices are not. Thus 𝑤 1 is a trail, but not a
path. 𝑤 2 is a walk where both the vertices and edges are distinct. Thus, 𝑤 2 is a path.
(40) Consider the following graph 𝐺:

List the components of 𝐺 − {𝑏 𝑓 } and 𝐺 − {𝑏, 𝑔}.


Solution. Notice that 𝐺 − {𝑏 𝑓 } is the edge induced subgraph obtained from 𝐺 by deleting
only the edge 𝑏 𝑓 (without deleting the vertices 𝑏 and 𝑓 ). Thus, 𝐺 − {𝑏 𝑓 } would have two
components as shown below.

12
On the other hand, 𝐺 − {𝑏, 𝑔} = 𝐺 [𝑉 (𝐺) − {𝑏, 𝑔}] = 𝐺 [{𝑎, 𝑐, 𝑑, 𝑒, 𝑓 , ℎ}] is the vertex
induced subgraph shown below:

(41) Find the adjacency matrix of the following graph:

Solution. Recall that the adjacency matrix of a graph 𝐺 is a square matrix of size
|𝑉 (𝐺)| × |𝑉 (𝐺)|. In this case, |𝑉 (𝐺)| = 4. If 𝑢 𝑎 and 𝑢 𝑏 are any two vertices of 𝐺. Then
(
#{edges of 𝐺 between 𝑢 𝑎 and 𝑢 𝑏 }, 𝑎 ≠ 𝑏,
[ 𝐴(𝐺)] 𝑎𝑏 =
2#{loops on 𝑢 𝑎 }, 𝑎 = 𝑏.
Thus
0 1 0 0

1 0 1 1
𝐴(𝐺) = 
0 1 0 0
0 1 0 2

(42) Find the adjacency matrix of 𝐾4 .
Solution. Recall that 𝐾4 is the unique complete (simple) graph with 4 vertices. Thus, we
have
0 1 1 1
 
1 0 1 1
𝐴(𝐾4 ) = 
 
 1 1 0 1 

1 1 1 0
 
13
(43) Note from Problem 42 that the sum of the entries Í inÍ𝐴(𝐾4 ) equals 12 which equals 2|𝐸 (𝐾4 )|.
Show that for any graph 𝐺 with 𝑛 vertices, 𝑛𝑎=1 𝑛𝑏=1 [ 𝐴(𝐺)] 𝑎𝑏 = 2|𝐸 (𝐺)|.
Solution. Let 𝑉 (𝐺) = {𝑢 1 , 𝑢 2 , · · · , 𝑢 𝑛 }. From handshaking lemma, we have
∑︁𝑛
𝑑 (𝑢 𝑎 ) = 2|𝐸 (𝐺)|. (2.1)
𝑎=1
Using the properties of row sums of 𝐴(𝐺), we find
𝑛
∑︁
[ 𝐴(𝐺)] 𝑎𝑏 = 𝑑 (𝑢 𝑎 ). (2.2)
𝑏=1
Using (2.2) in (2.1) yields
𝑛 ∑︁
∑︁ 𝑛
[ 𝐴(𝐺)] 𝑎𝑏 = 2|𝐸 (𝐺)|.
𝑎=1 𝑏=1

14

You might also like